-1
$\begingroup$

Let $S_n$ be defined as $\frac{1}{n}\sum_{t=1}^{t=n} [px_t^2 - (p+q)x_t]$ where $x_t = 1-(1-p-q)^t$. We want to find the conditions on $p$ and $q$ such that $S_n$ is monotonically decreasing for all $n$. $0 < p,q < 1$ and $-1 < 1-p-q < 1$.

Note: I tried to prove this by taking the difference between consecutive terms of the summand i.e. $[px_t^2 - (p+q)x_t] - [px_{t+1}^2 - (p+q)x_{t+1}]]$ and trying to show that it is greater than 0 but I am not able to make progress on this.

$\endgroup$

1 Answer 1

1
$\begingroup$

Since $x_0=0$, it will be convenient to do summation starting from $t=0$.

Denoting $r:=1-p-q$, we have \begin{split} S_n &= \frac1n\sum_{t=0}^n \left(pr^{2t} + (q-p)r^{t} - q\right) \\ &=\frac{p}{1-r^2} \frac{1-r^{2(n+1)}}{n} + \frac{q-p}{1-r} \frac{1-r^{n+1}}{n} - \frac{n+1}{n}q. \end{split}

Given that $|r|<1$, it can be easily verified that $$\frac{\partial S_n}{\partial n} = \frac{(p - (r + 1) q)r}{n^2(1-r^2)} + O(r^n)$$ as $n$ grows.

Hence, $S_n$ decreases for large enough $n$ iff $p-(r+1)q\lessgtr 0$ or $$p\lessgtr\frac{(2-q)q}{1+q},$$ where the inequality sign is "$<$" if $r>0$, and "$>$" if $r<0$. In other words, either of the following two conditions works: $$p < \min\left\{\frac{(2-q)q}{1+q},1-q\right\}$$ or $$p > \max\left\{\frac{(2-q)q}{1+q},1-q\right\}.$$

$\endgroup$
2
  • $\begingroup$ Thanks for the answer! So $S_n$ will decrease in the long run but we cannot say anything about it when $n$ is small? $\endgroup$ Sep 13, 2020 at 0:34
  • $\begingroup$ You'd need to take a close look at $\frac{\partial S_n}{\partial n}$ to make conclusion about the behavior of $S_n$ for small $n$. At very least, the two conditions I derived are necessary for $S_n$ to decrease. $\endgroup$ Sep 13, 2020 at 2:57

Your Answer

By clicking “Post Your Answer”, you agree to our terms of service and acknowledge that you have read and understand our privacy policy and code of conduct.

Not the answer you're looking for? Browse other questions tagged or ask your own question.